#P1249D2. Too Many Segments (hard version)

Too Many Segments (hard version)

Description

The only difference between easy and hard versions is constraints.

You are given $n$ segments on the coordinate axis $OX$. Segments can intersect, lie inside each other and even coincide. The $i$-th segment is $[l_i; r_i]$ ($l_i \le r_i$) and it covers all integer points $j$ such that $l_i \le j \le r_i$.

The integer point is called bad if it is covered by strictly more than $k$ segments.

Your task is to remove the minimum number of segments so that there are no bad points at all.

The first line of the input contains two integers $n$ and $k$ ($1 \le k \le n \le 2 \cdot 10^5$) — the number of segments and the maximum number of segments by which each integer point can be covered.

The next $n$ lines contain segments. The $i$-th line contains two integers $l_i$ and $r_i$ ($1 \le l_i \le r_i \le 2 \cdot 10^5$) — the endpoints of the $i$-th segment.

In the first line print one integer $m$ ($0 \le m \le n$) — the minimum number of segments you need to remove so that there are no bad points.

In the second line print $m$ distinct integers $p_1, p_2, \dots, p_m$ ($1 \le p_i \le n$) — indices of segments you remove in any order. If there are multiple answers, you can print any of them.

Input

The first line of the input contains two integers $n$ and $k$ ($1 \le k \le n \le 2 \cdot 10^5$) — the number of segments and the maximum number of segments by which each integer point can be covered.

The next $n$ lines contain segments. The $i$-th line contains two integers $l_i$ and $r_i$ ($1 \le l_i \le r_i \le 2 \cdot 10^5$) — the endpoints of the $i$-th segment.

Output

In the first line print one integer $m$ ($0 \le m \le n$) — the minimum number of segments you need to remove so that there are no bad points.

In the second line print $m$ distinct integers $p_1, p_2, \dots, p_m$ ($1 \le p_i \le n$) — indices of segments you remove in any order. If there are multiple answers, you can print any of them.

Samples

7 2
11 11
9 11
7 8
8 9
7 8
9 11
7 9
3
4 6 7
5 1
29 30
30 30
29 29
28 30
30 30
3
1 4 5
6 1
2 3
3 3
2 3
2 2
2 3
2 3
4
1 3 5 6